0% found this document useful (0 votes)
256 views7 pages

Functional 3

This document appears to be an assignment submission containing 10 functional analysis problems and their solutions. It was submitted on July 4, 2016 to Dr. Shiferaw Feyissa by 5 students, including Abener Tewodros, Hana Endiris, Habtamu Meressa, Miliyon Tilahun, and Sisai Bekele. The problems cover topics such as Banach spaces, continuous linear operators, closed subspaces, Hilbert spaces, and contractions.

Uploaded by

Miliyon Tilahun
Copyright
© © All Rights Reserved
Available Formats
Download as PDF, TXT or read online on Scribd
Download as pdf or txt
0% found this document useful (0 votes)
256 views7 pages

Functional 3

This document appears to be an assignment submission containing 10 functional analysis problems and their solutions. It was submitted on July 4, 2016 to Dr. Shiferaw Feyissa by 5 students, including Abener Tewodros, Hana Endiris, Habtamu Meressa, Miliyon Tilahun, and Sisai Bekele. The problems cover topics such as Banach spaces, continuous linear operators, closed subspaces, Hilbert spaces, and contractions.

Uploaded by

Miliyon Tilahun
Copyright
© © All Rights Reserved
Available Formats
Download as PDF, TXT or read online on Scribd
Download as pdf or txt
Download as pdf or txt
You are on page 1/ 7

Assignment III

FUNCTIONAL ANALYSIS


July 4, 2016

Submitted to

Dr. Shiferaw Feyissa

Name
1.
2.
3.
4.
5.

Abener Tewodros .
Hana Endiris . . .
Habtamu Meressa
Miliyon Tilahun . .
Sisai Bekele . . . .

.
.
.
.
.

.
.
.
.
.

.
.
.
.
.

.
.
.
.
.

.
.
.
.
.

.
.
.
.
.

.
.
.
.
.

.
.
.
.
.

.
.
.
.
.

.
.
.
.
.

.
.
.
.
.

.
.
.
.
.

.
.
.
.
.

.
.
.
.
.

.
.
.
.
.

.
.
.
.
.

.
.
.
.
.

.
.
.
.
.

.
.
.
.
.

.
.
.
.
.

.
.
.
.
.

ID No
. GSR/1417/08
. GSR/1419/08
. GSR/1399/08
. GSR/1401/08
. GSR/1426/08

Functional Analysis Problems


1. Let X be a finite dimensional normed space and Y be normed space.
Show that
(a) X is a Banach space.
(b) Every linear operator T : X Y is continuous.
Proof. (a) Suppose (X, k k) is the normed space, and (ei )N
i=1 is a basis
PN
for X. For x = j=1 j ej , we can then define
v
uN
uX
0
kxk = t
|j |2
j=1

whence k k0 : X R is a norm for X. Since all norms on a finite


dimensional normed space are equivalent, there is a constant C > 0
such that
1
kxk0 kxk Ckxk0 , x X.
C
To prove that X is a Banach space, let x1 , x2 , . . . be a Cauchy sequence
in (X, k k). That is, for all > 0 there is an M 1 such that
kxj xk k < , for all j, k M.
Let us write each xk in this sequence in the basis (ej ) as xk =
for some constants k,j C. For k, l 1 we then have
kxk xl k

PN

j=1

k,j ej

1
kxk xl k0
Cv
uN
X
1u
t
|k,j l,j |2
C j=1

1
|k,j l,j |
C

for all j = 1, . . . , N . It follows that (k,1 )


k=1 , . . . , (k,N )k=1 are Cauchy
sequences in C. As C is complete,
these converge to some complex
PN
numbers 1 , . . . , N . Let x = j=1 j ej .

For each k = 1, 2, . . ., we then have


kx xk k

Ckx xk k0
v
uN
uX
|j k,j |2 .
Ct
j=1

By taking k it follows that (xj ) converges to x V .


(b) Let e1 , . . . , en be a basis of X. Then for x X we have
X
X
X
kT xkY = kT
i ei kY = k
i T ei kY
|i |kT ei kY
i

Let > 0 be given and let M = maxi kT ei kY . Let = M


. Then for x
with kxk, we have
X
X
kT xkY
|i |kT ei kY M
|i | <
i

Hence T is continuous.
R1
2. Let X = C[0, 1] with norm kf k1 = 0 |f (t)|dt and Y = C[0, 1] with
norm kf k = supt[0,1] |f (t)|. Define K : C[0, 1] C[0, 1] R by
R1
K(f ) = 0 K(s, t)f (t)dt, s [0, 1]. Show that K is a continuous linear
map.
Solution.

3. Let T : X R be a linear functional such that T is non-zero. Show


that the following statements are equivalent.
(a) T is continuous.
(b) The nullity of T is closed.
Solution. (a b) If T is continuous, then the inverse image of closed
set under T is closed. Hence
ker T = T 1 [{0}]
is closed.
(a b) Assume T 6= 0 and T 1 [{0}] is closed. Pick e X such that
f (e) = 1.
Suppose by way of contradiction kT k = . Then there exists a sequence
{xn } in X with kxn k = 1 and T (xn ) n for all n. Note that the
xn
sequence yn defined by yn = e T (x
, satisfies yn T 1 [{0}] for all n
n)
and yn e. Since the set is closed it follows that e must belong to it
and consequently T (e) = 0 which is a contradiction.
4. Let c = {xn } be the set of all sequences in R such that limn xn
exists. Show that c is a closed subspace of ` and hence is a Banach
space.
(n)

Solution. For each n, let n = {xk }k0 is in c and hence convergent.


(n)
Let say that n = {xk }k0 converges to un R. Now
|um un | km n k ,

m, n N

So since {n }n0 is a Cauchy sequence in c, it follows that {un }n0 is a


Cauchy sequence in R, so there exists u R such that limn un = u.
Claim = {xk }k0 converges to u. To see this
(n)

(n)

|xk u| |xk xk | + |xk un | + |un u|


(n)

k n k + |xk un | + |un u|
Let > 0. Choose n0 N such that |un0 u| < 3 and k n0 k < 3 .
(n )
Choose K N such that |xk 0 un0 | < 3 whenever k > K. Then for
k K all three terms on the right hand side of the above equation are
less than 3 , and thus |xk u| < .
Thus xk u. Therefore = {xk }k0 converges so c and hence c is
a Banach subspace of ` .

5. Show that the dual space of `p is `q where

1
p

1
q

= 1.

Solution.
6. Let T be a self adjoint bounded linear operator in a Hilbert space H.
Prove that
kT k = sup |hx, T (x)i|
kxk=1

Solution.
7. Let H be a Hilbert space and fy (x) = hy, xi be a bounded linear functional. Prove that kfy k = kyk.
Solution.

8. Show that T : R R defined by T (x) = 2 + x tan1 x has no fixed


point and |T (x) T (y)| < |x y| for all x, y R.
Solution. We have

T x x = + x arctan(x) x
2

= arctan(x) > 0
(since /2 < arctan(x) < /2)
2
So T has no fixed point.
Since
T 0 (x) =

x2
1 + x2

we have 0 < T 0 < 1. The the Mean Value Theorem implies that if x 6= y
T x T y = T 0 ()(x y)
for some between x and y, so
|T x T y| < |x y|

9. Let X and Y be normed space. If T1 : X Y is a closed linear operator


and T2 : X Y be a bounded linear operator then show that T1 + T2
is closed linear operator.
Solution. Let xn X, and x X, y Y such that as n
xn x in X
and
(T1 + T2 )xn = T1 xn + T2 xn y

in Y

Since T2 is bounded there exists


T2 x = lim T2 xn
n

and so T1 xn y T2 x.
The closedness of T1 implies that x X and T1 x = y T2 x that is
(T1 + T2 )x = T1 x + T2 x = y.

10. If T is a contraction then show that T n where n is a positive integer is


a contraction. If T n is a contraction for n > 1 show that T may not be
a contraction.
Solution.

You might also like